1
$\begingroup$

Let G be the node-arc incidence matrix of a given directed network (rows of $G$ correspond to nodes and its columns correspond to arcs). Let $B_1,\dots, B_K$ denote a partition of the nodes of the network. Suppose the network is such that a directed arc can go from a node in $B_k$ to another node in $B_\ell$ only if $k<\ell$. Let $H$ denote a matrix with $K$ rows and suppose that its columns are indexed by the arcs of the underlying network. We assume that $H$ is such that its $(k,e)$-th entry is equal to one if $e\in B_k$, and zero otherwise.

Is the matrix [H;G] (obtained by the concatenation of rows of $H$ and $G$) totally unimodular? If not, can you give a counter example?

I've explored a few examples numerically, and verified total unimodularity for these examples. I thought it may be possible the exploit the structure of $H$ (note the special row structure) to prove the result formally. I've tried leveraging the Ghouila-Houri condition (see https://en.wikipedia.org/wiki/Unimodular_matrix) which seems like a suitable candidate for exploiting the row structure. But I was not successful so far.

$\endgroup$
2
  • $\begingroup$ X-posted: math.stackexchange.com/q/3437674/339790 $\endgroup$ Commented Nov 17, 2019 at 7:59
  • $\begingroup$ I’m a little confused what is [H;G]. Do you mean that you concatenate every row of H with every row of G? Or do you mean to use H transpose? $\endgroup$ Commented Nov 18, 2019 at 1:29

0

You must log in to answer this question.

Start asking to get answers

Find the answer to your question by asking.

Ask question

Explore related questions

See similar questions with these tags.